User avatar
 
noah
Thanks Received: 1192
Atticus Finch
Atticus Finch
 
Posts: 1541
Joined: February 11th, 2009
 
 
 

Logic Challenge #31 - The Presentation Game (Hard)

by noah Mon Apr 25, 2011 12:41 pm

Please post your explanations here for the hard version of the challenge. If you want to attach a pdf, shoot it over to us at studentservices(@)manhattanlsat.com (the parentheses here help us control spam).

Good luck!
 
rpcuhk
Thanks Received: 5
Forum Guests
 
Posts: 41
Joined: May 02nd, 2011
 
This post thanked 1 time.
 
 

Re: Logic Challenge #31 - The Presentation Game (Hard)

by rpcuhk Mon May 02, 2011 3:08 pm

Hi Noah, I have emailed my explanation to the said address.
Thanks!

And here it is!
the-presentation-game.pdf
(73.84 KiB) Downloaded 917 times
User avatar
 
ManhattanPrepLSAT2
Thanks Received: 311
Atticus Finch
Atticus Finch
 
Posts: 303
Joined: July 14th, 2009
 
 
 

Re: Logic Challenge #31 - The Presentation Game (Hard)

by ManhattanPrepLSAT2 Mon Jun 13, 2011 5:26 pm

Terrific explanation rpcuhk! Thanks so much for contributing to our forums, and I hope you found the exercise of writing the solution to be a helpful learning experience. The explanation makes it pretty clear that you are a games jedi!

If you have an additional second, or if anyone else wants to chime in, I would love to see a little more elaboration about the final rule equivalency question. I'm glad you nailed it, but I know it threw many people for a loop, and it might prove useful to explain why some of those other choices are incorrect.

Thanks again for your great work! Good luck on the next game. :) -- MK
 
rpcuhk
Thanks Received: 5
Forum Guests
 
Posts: 41
Joined: May 02nd, 2011
 
This post thanked 1 time.
 
 

Re: Logic Challenge #31 - The Presentation Game (Hard)

by rpcuhk Tue Jun 14, 2011 11:39 pm

Thank you Mike!
Let me try my hand at the wrong answers.
Answer choice (A) gets us the following conditional statement,
"If L is before P, then M is also before P"
but we know that when L is before P, M is after P, so (A) is incorrect.
Answer choice (B) is in agreement with the condition we are trying to replace, but is not quite the same.
Because the original condition is a if and only if statement, it means the conditional statement in B only tells us half the story.
Answer choice (C) is wrong for the same reason that makes answer choice (B) wrong.
Answer choice (E) is wrong for the same reason that makes answer choice (A) wrong.







Mike.Kim Wrote:Terrific explanation rpcuhk! Thanks so much for contributing to our forums, and I hope you found the exercise of writing the solution to be a helpful learning experience. The explanation makes it pretty clear that you are a games jedi!

If you have an additional second, or if anyone else wants to chime in, I would love to see a little more elaboration about the final rule equivalency question. I'm glad you nailed it, but I know it threw many people for a loop, and it might prove useful to explain why some of those other choices are incorrect.

Thanks again for your great work! Good luck on the next game. :) -- MK
User avatar
 
ManhattanPrepLSAT2
Thanks Received: 311
Atticus Finch
Atticus Finch
 
Posts: 303
Joined: July 14th, 2009
 
 
 

Re: Logic Challenge #31 - The Presentation Game (Hard)

by ManhattanPrepLSAT2 Wed Jun 15, 2011 5:24 pm

Awesome -- thanks so much for the further explanation!
 
siliconrs
Thanks Received: 1
Forum Guests
 
Posts: 7
Joined: November 06th, 2010
 
 
 

Re: Logic Challenge #31 - The Presentation Game (Hard)

by siliconrs Tue Jul 19, 2011 4:28 pm

rpcuhk Wrote:Hi Noah, I have emailed my explanation to the said address.
Thanks!

And here it is!
the-presentation-game.pdf


In your PDF, the last answer to the last question is missing a word or two.
 
fyami001
Thanks Received: 0
Vinny Gambini
Vinny Gambini
 
Posts: 19
Joined: May 08th, 2011
 
 
 

Re: Logic Challenge #31 - The Presentation Game (Hard)

by fyami001 Fri Jul 06, 2012 8:21 pm

Hi! :D
I don't find the solution to number 5 in the pdf satisfactory. It seems to me that O is a floater in Option 1, so putting M first would not make it the same as option 2. It would be O-N-L..etc. The same thing goes for B and D. The only ones we can eliminate for sure are C and E.

Please Help as soon as possible, I'm doing this on my friday night lol
User avatar
 
ohthatpatrick
Thanks Received: 3808
Atticus Finch
Atticus Finch
 
Posts: 4661
Joined: April 01st, 2011
 
 
 

Re: Logic Challenge #31 - The Presentation Game (Hard)

by ohthatpatrick Thu Jul 12, 2012 3:03 am

Well, I'm sure this is reaching you later than you hoped .... I just got assigned the case today. :)

What the writer of that pdf was trying to say is that L--P only happens in Option 1.

So as we examine the answer choices on Q5, we know that if an answer choice FORCES us to use Option 1, then we know that L -- P must be true.

If an answer choice does NOT force us to use Option 1 (if we could also use Option 2), then L -- P doesn't HAVE to be true.

A) O = 1
Does that FORCE us to use option 1? No, O could be 1 in either Option 1 or Option 2.

I'll write a quick scenario from each Option
1: O N L P K M
2: O M N P K L

Clearly, putting O in spot 1 does not GUARANTEE that L is before P.

B) O = 2
Same question. Does that FORCE us to use option 1? No, either works.
1: N O L P K M
2: N O M P K L

So putting O in spot 2 does not GUARANTEE that L is before P.

C) O = 3
Does that FORCE us to use option 1? No, in fact, it's impossible to use either scenario because this breaks a rule.

D) O = 4
Does this FORCE us to use option 1? Yes.

In Option 2, we could never put O in spot 4. In Option 2, M/P/L all come after O, so putting O in spot 4 wouldn't leave room for M, P, and L.

We are forced to use option 2.
2: N L P O (K M) (last two are interchangeable)

E) As we've all noted, this is impossible in either Option.

Hope this helps.
 
anne.monjar
Thanks Received: 0
Vinny Gambini
Vinny Gambini
 
Posts: 3
Joined: January 29th, 2015
 
 
 

Re: Logic Challenge #31 - The Presentation Game (Hard)

by anne.monjar Mon Apr 06, 2015 9:00 pm

Hi all -

Confused by question 7. See why answer D is correct, but don't see how choices B and C aren't exactly the same.

Does B translate to LP --> PM (and thus, by contrapositive, MP --> PL) ?

And similarly C - wouldn't that translate to MP -- > PL (and contrapositive LP --> PM) ?

How are either of those options different from "either L or M, but not both, are before P?"

Any guidance appreciated!

Annie
User avatar
 
ohthatpatrick
Thanks Received: 3808
Atticus Finch
Atticus Finch
 
Posts: 4661
Joined: April 01st, 2011
 
 
 

Re: Logic Challenge #31 - The Presentation Game (Hard)

by ohthatpatrick Tue Apr 14, 2015 9:24 pm

The original rule is basically
L/M - P - M/L

(B) says
L - P --> P - M
contrapositive
M - P --> P - L

But ... this still allows for P to be before both of them.
PLM does not break the rule in (B), though it does break the original rule.

Neither version of the rule in (B) is triggered by PLM, so the rule is compatible with PLM.

(C) says
M - P --> P - L (this is identical to B)
contrapositive
L - P --> P - M

It's the exact same rule, so the same PLM exception is what's killing it.

Hope this helps.